Đến nội dung

Kamii0909 nội dung

Có 155 mục bởi Kamii0909 (Tìm giới hạn từ 28-04-2020)



Sắp theo                Sắp xếp  

#689342 Đề Thi Trại Hè Hùng Vương 2017

Đã gửi bởi Kamii0909 on 03-08-2017 - 00:23 trong Thi HSG cấp Tỉnh, Thành phố. Olympic 30-4. Đề thi và kiểm tra đội tuyển các cấp.

Bài 4.
$a+b^2 \mid a^2+b$ nên $a \geq b$ 

$a=b$ thì $2 \mid a(a+1)$, do đó $2^x=a(a+1)$
Nếu $a\geq 2$ thì $a(a+1)$ sẽ có ước khác 2(vô lý)
Vậy $a=b=1$
Xét $a > b \geq 1$
$a+b^2 \mid a^2+b$
$\Leftrightarrow a+b^2 \mid a(a+b^2)-b(ab-1)$
Mà $(a,ab-1)=1$ nên $a+b^2 \mid b(a+b^2)-b^3-1$
Hay $p^x=a+b^2 \mid (b+1)(b^2-b+1)$
Dễ thấy $a+b^2 >b+1$ và $a+b^2>b^2-b+1$ nên $p|(b+1,b^2-b+1)$
Từ đó có $p=3$
Mà $9 \nmid b^2-b+1$ nên $3^{x-1} \mid b+1$
$\Rightarrow b \geq 3^{x-1}-1 = \dfrac{a+b^2}{3} -1$
Hiển nhiên điều này sai với $b \geq 3$, mà $3^{x-1} \mid b+1$ nên $b=2, a=5$




#689899 Đề Thi Trại Hè Hùng Vương 2017

Đã gửi bởi Kamii0909 on 08-08-2017 - 13:14 trong Thi HSG cấp Tỉnh, Thành phố. Olympic 30-4. Đề thi và kiểm tra đội tuyển các cấp.

cách này mình đã post thì bạn/anh/chị post lại làm gì nữa

$p$ chỉ có max là $(b+1,b^2-b+1)$ và k có điều kiện gì để suy ra được nó là ước cả , cho nên xét thiếu $p=2$ r kìa 

chọn $b=3$ và $a=1$ thì $b=3 \geq \frac{a+b^2}{3}-1=\frac{7}{3}$

$2 \nmid b(b-1)+1$
$a>b$




#693450 Đề thi chọn học sinh giỏi THPT Khoa Học Tự Nhiên 2017-2018

Đã gửi bởi Kamii0909 on 21-09-2017 - 00:48 trong Thi HSG cấp Tỉnh, Thành phố. Olympic 30-4. Đề thi và kiểm tra đội tuyển các cấp.

Bài 2: Tìm tất cả các đa thức $P(x)$ hệ số nguyên không âm thoả mãn $P(\sqrt [3]{3})=2017$ và $P(1)$ nhận giá trị nhỏ nhất có thể.

 

Bài 7: Cho a,b,c là các số thực thỏa mãn $(a+b)(b+c)(c+a) ≠ 0$. Chứng minh rằng:

$\frac {(a^2-b^2)(a^2-c^2)}{(b+c)^2} + \frac {(b^2-c^2)(b^2-a^2)}{(c+a)^2} + \frac {(c^2-a^2)(c^2-b^2)}{(a+b)^2} \geq 0$

Bài 2. 

Dễ thấy $P(x)=(x^3-3)Q(x)+2017$

Đặt $Q(x)=a_{n} x^n+a_{n-1} x^{n-1}+...+a_0$ 

$P(x)=a_{n} x^{n+3}+a_{n-1} x^{n+2} +a_{n-2} x^{n+1} +( a_{n-3}-3a_{n} ) x^n+(a_{n-4}-3a_{n-1}) x^{n-1}+...+(a_{0}-3a_{3})x^3-3a_{2} x^2 -3a_{1} x +2017 - 3a_{0}$

Do $P(x)$ có hệ số không âm nên ta phải có hệ
$$\left\{\begin{matrix} a_{n},a_{n-1},a_{n-2} \geq 0\\ a_{n-3} \geq 3a_{n}\geq 0\\ ...\\ a_{0} \geq 3 a_{3} \geq 0\\ a_1 ,a_2 \leq 0\\ a_0 \leq \dfrac{2017}{3}\\ \end{matrix}\right.$$

Cho ta các nghiệm nguyên không âm $a_{n}=a_{n-1}=...=a_{1}=0$ hay $Q(x)=a_0=c \leq 672$ là hàm hằng. 

$P(1)=c+2017-3c=2017-2c \geq 673$

Dấu "=" xảy ra khi $P(x)=672 x^3 +1$

Bài 7. 

$VT=f(a,b,c) \geq f(|a|,|b|,|c|)$nên ta chỉ cần chứng minh trong TH a,b,c không âm. 

KMTTQ, $a \geq b \geq c$

$f(a,b,c)=\sum \dfrac{(a-b)(a-c)(a+b)(a+c)}{(b+c)^2}$

Dễ thấy $\dfrac{(a+b)(a+c)}{(b+c)^2} \geq \dfrac{(b+a)(b+c)}{(c+a)^2}$ 

nên theo bất đẳng thức Vornicul-Schur ta có đpcm.

 

#Ps: 2 bài hình vòng 1 năm nay có vẻ không thấm lắm :V 




#657034 $\sum \frac{4}{a+b} \leq \frac{1}{a}+\frac{1}{b...

Đã gửi bởi Kamii0909 on 07-10-2016 - 20:40 trong Bất đẳng thức và cực trị

Bài 6 $2ab+6bc+2ac=7abc$ <=> $\frac{2}{c}+\frac{6}{a}+\frac{2}{b}=7\Leftrightarrow 6x+2y+2z=7$

C=$\frac{4}{y+2x}+\frac{9}{z+4x}+\frac{4}{y+z}\geq \frac{\left (2+3+2 \right )^{2}}{6x+2y+2z}=7$ 




#658044 $\sum\sqrt[3]{\frac{c}{b+a}}>\frac{\sqrt[3]...

Đã gửi bởi Kamii0909 on 16-10-2016 - 11:33 trong Bất đẳng thức và cực trị

1/ Cho a,b,c>0 thỏa mãn: $\frac{1}{1+a}+\frac{1}{1+b}+\frac{1}{1+c}\geq 2$CMR: $abc\leq \frac{1}{8}$
2/ Cho a,b >0, thỏa mãn a+b=1. CMR: $(a+\frac{1}{a})^2+(b+\frac{1}{b})^2\geq \frac{25}{2}$
3/ Cho a,b,c >0 thỏa mãn: a+b+c+d=1. CMR:
$(a+\frac{1}{a})^2+(b+\frac{1}{b})^2+(c+\frac{1}{c})^2+(d+\frac{1}{d})^2\geq \frac{289}{4}$
4/ Cho a,b,c >0. CMR: $\sqrt[3]{\frac{a}{b+c}}+\sqrt[3]{\frac{b}{a+c}}+\sqrt[3]{\frac{c}{b+a}}>\frac{\sqrt[3]{2}}{2}$

Ta có bất đẳng thức sau với mọi a,b,c không âm

$\sum \sqrt[3]{\frac{a}{b+c}}\geq \sum \sqrt{\frac{a}{b+c}}\geq 2$

Dấu bằng xảy ra khi 1 biến =0 và 2 biến còn lại bằng nhau




#693445 Turkey TST 2017

Đã gửi bởi Kamii0909 on 20-09-2017 - 23:17 trong Thi HSG Quốc gia và Quốc tế

 

$f$ toàn ánh . Gọi $P(x_0,y_0)$ là phép thế 
Nếu $f(x_1) = f(x_2) $ xét $P(x_1,x_2)$ và $P(x_2,x_1)$ có $x_1 = x_2 \implies f$ đơn ánh $\implies f$ toàn ánh
$P(0,y) : f(f(y)) = by+a $ với $b = f(0) $ , $P(x,f(y)) : f(xf(y)+by+a) = f(x)f(y)+a = f(yf(x)+bx+a) \implies xf(y)+by = yf(x)+bx \implies f(x) = mn+x$  Thế vào thì có $m^2 = n $ và $m^3+m^2 = a$
Vậy $f(x) = mx+m^2$ với $m$ là nghiệm của phương trình $x^3+x^2 - a = 0$ 

 

Đoạn cm đơn ánh chưa hoàn chỉnh. Nếu $f(x_1)=f(x_2)=0$ thì $x_1 \neq x_2$ thoải mái. TH $a=0$ có $f(x)=0, \forall x$ chứng tỏ điều này. 
Có thể xử lí kiểu này. 
$P(0,x):f(f(x))=xf(0)+a$

Nếu $f(0)=0$ thì $P(x,0):a=0$

Hay $P(0,x):f(f(x))=0$

Nếu $f$ hằng thì $f(x)=0,\forall x$. 

Nếu $f$ không hằng thì $\exists c,f(c) \neq 0$ 
$P(c,x):f(cx+f(x))=xf(c)$ nên $f$ toàn ánh. 
Do đó $P(0,x)$ sẽ cho ta $f(x)=0,\forall x$(là hàm hằng, mâu thuẫn) 

 

Xét $f(0) \neq 0$ thì $P(0,x)$ cho $f$ song ánh và có thể tiếp tục giải như trên.




#675568 $\prod (a^{2}+b^{2})\leq \frac{1...

Đã gửi bởi Kamii0909 on 28-03-2017 - 22:46 trong Bất đẳng thức - Cực trị

Cmr $$f(a,b,c) \leq f(a+b,c,0)$$.



#676603 $\prod (a^{2}+b^{2})\leq \frac{1...

Đã gửi bởi Kamii0909 on 08-04-2017 - 00:01 trong Bất đẳng thức - Cực trị

Biểu thức $f(a,b,c)$ của em là gì ? Nếu dồn biến theo kiểu này thì sẽ chọn $c$ là số nhỏ nhất, anh thử nhẩm với $f(a,b,c) = (a^2+b^2)(b^2+c^2)(c^2+a^2)$ khi xét $f(a,b,c) \leqslant f(a+b,c,0)$ thì hai đại lượng trội nhất là $a^3b,ab^3$ nằm bên trái dấu $\leqslant $ nên có thể bất đẳng thức này sai.

Em chọn $c$ là số lớn nhất.
Ta nhân 2 đánh giá sau
$c^2[(a+b)^2+c^2]=c^4+c^2a^2+c^2b^2+2c^2ab \geq (c^2+a^2)(c^2+b^2)$
Và $(a+b)^2 \geq (a^2+b^2)$



#676639 $\prod (a^{2}+b^{2})\leq \frac{1...

Đã gửi bởi Kamii0909 on 08-04-2017 - 18:51 trong Bất đẳng thức - Cực trị

Nhưng như vầy thì em dồn $c$ về $0$ !

Em chưa hiểu ý anh lắm. Em đang dồn $b \rightarrow 0$. Nếu dồn $c \rightarrow 0$ thì có lẽ là $f(a+c,b,0)$ chính xác hơn.



#676727 $\prod (a^{2}+b^{2})\leq \frac{1...

Đã gửi bởi Kamii0909 on 09-04-2017 - 13:15 trong Bất đẳng thức - Cực trị

Em định nghĩa $f(a,b,c) = (a^2+b^2)(b^2+c^2)(c^2+a^2)$ thì $f(a+b,c,0)$ tức là thay $a = a + b,\, b = c$ và $c = 0.$ Tức $c$ là nhỏ nhất.

Thực ra nó chỉ là mặt quy ước, ta có thể thấy $f(a+c,b,0)$ và $f(a+b,c,0)$ hay $f(0,b,a+c)$ là tương đương hết sau phép đặt ẩn $(a+c,b) \rightarrow (x,y)$ hay $(a+b,c) \rightarrow (x,y)$
Ta chọn vị trí các số sao cho chứng minh dễ nhất có thể mà thôi. Nó không ảnh hưởng dù $c$ max hay min. Bài toán này có thể chọn thứ tự thoải mái do tính đối xứng.



#670334 $\frac{a}{a^2+1}+\frac{b}{b^2+1}+\frac{c}{c^2+1}\leq...

Đã gửi bởi Kamii0909 on 29-01-2017 - 13:19 trong Bất đẳng thức và cực trị

CM tương đương.
chuyển vế: $(\frac{a}{a^2+1}-\frac{3}{10})+(\frac{b}{b^2+1}-\frac{3}{10})+(\frac{c}{c^2+1}-\frac{3}{10})\leq 0<=>\sum( \frac{-3a^2+10a-3}{a^2+1})$(luôn đúng).
Dấu "=" xảy ra khi x=y=3.

Hãy đánh giá thử bất đẳng thức cuối xem vì sao nó đúng?

$$\sum \dfrac{a}{a^2+1} \leq \frac{9}{10} $$
$$\Leftrightarrow \sum \dfrac{a}{a^2+1} \leq \frac{9}{10} + \sum \dfrac{6(3a-1)}{25}$$
$$\Leftrightarrow \sum \dfrac{(3a-1)^2(4a+3)}{50(a^2+1)}\geq 0$$
Hiển nhiên đúng.



#658292 ĐỀ THI HSG THPT CHUYÊN VÀ CHỌN ĐỘI TUYỂN DỰ THI HSG QUỐC GIA THPT NĂM 2016-20...

Đã gửi bởi Kamii0909 on 18-10-2016 - 16:09 trong Thi HSG cấp Tỉnh, Thành phố. Olympic 30-4. Đề thi và kiểm tra đội tuyển các cấp.

 

ĐỀ THI HSG THPT CHUYÊN VÀ CHỌN ĐỘI TUYỂN DỰ THI HSG QUỐC GIA THPT NĂM 2016-2017 TỈNH QUẢNG NAM

 

 

Chứng minh bất đẳng thức: $ (a+b+c+d)^3 \le 4(a^3+b^3+c^3) +24(abc+bcd+cda+dab) $

 

Thiếu $d^{3}$ nữa chứ nhỉ  :icon6:  :icon6:




#693813 Đề chọn đội tuyển QG Dak Lak năm 2017-2018

Đã gửi bởi Kamii0909 on 27-09-2017 - 20:40 trong Thi HSG cấp Tỉnh, Thành phố. Olympic 30-4. Đề thi và kiểm tra đội tuyển các cấp.

Bài 5. 

Chứng minh được

$$x_{n}+x_{n-1}=\dfrac{-1}{2017^{n-1}}$$

Từ đó suy ra CTTQ 

$$x_{n}=\dfrac{2017 \cdot (-1)^n}{2018}- \dfrac{1}{2018 \cdot 2017^{n-1}}$$

Do đó $$\displaystyle \lim_{n \to + \inf} x_{n} =\dfrac{2017^2}{2018^2}$$

 

Bài 6. 

Từ giả thiết dễ có

$$f(x) \geq 2xy-f(y), \forall x,y$$

Cho $x=y$ thì $f(x) \geq x^2, \forall x$

Cố định $x$, chọn $y$ sao cho $2xy-f(y)$ lớn nhất. 

Khi đó $$f(x)=2xy-f(y) \leq 2xy-y^2 \leq x^2$$

Do đó $f(x)=x^2, \forall x$

Thử lại TM, kết luận...




#664443 Tìm số tự nhiên n>1 nhỏ nhất để $(n+1)(2n+1)\vdots 6$ và t...

Đã gửi bởi Kamii0909 on 12-12-2016 - 11:02 trong Số học

Đóng góp 1 cách cho bài 2.
Giả sử tồn tại số nguyên tố $p$ sao cho $p=a^n +b^n +c^n +d^n$. Do $ab=cd$ nên ta viết lại
$p= a^n +b^n +c^n +\frac{a^n.b^n}{c^n} \\ \Leftrightarrow p.c^n=(c^n+a^n)(c^n+b^n)$
Nên 1 trong 2 số $c^n+a^n,c^n+b^n$ phải là bội của $p$ mà rõ ràng 2 số này $<p$ nên mâu thuẫn. Vậy ta có đpcm.



#719502 $VMO2019$

Đã gửi bởi Kamii0909 on 14-01-2019 - 22:55 trong Thi HSG Quốc gia và Quốc tế

Topic ảm đạm quá mình chém bài đa thức vậy.

Bổ đề 1: $\Gamma(f(x))$ là hệ số tự do của $f(x)f(\dfrac{1}{x})$

Chỉ viết $f(x)$ và nhân ra thôi.

Bổ đề 2: Cho

$f(x)=a_{0}+...+a_{n} x^n$

$g(x)=b_{0}+...+b_{n} x^n$

$h(x)=f(x)(b_{0} x^n+...+b_{n})=f(x)x^n g(\dfrac{1}{x})$ (đảo hệ số của $g(x)$)

Thì $\Gamma(f(x)g(x)) = \Gamma(h(x))$ 

Chú ý $h(x)h(\dfrac{1}{x})=f(x)g(x)f(\dfrac{1}{x})g(\dfrac{1}{x})$ 

Quay lại bài toán

Với $n=1010$

Viết $P(x)=(x+a_{1})(x+a_{2})...(x+a_{n})(x+b_{1})(x+b_{2})...(x+b_{n})$

Trong đó $A \cap B = \{1,2,...,2n\}$

Số đa thức $Q_{k} (x)$ phân biệt tạo thành theo bổ đề 2 sẽ bằng vào số bộ phân biệt $b_{1}<b_{2}<...<b_{n}$ mà $b_{i} \in \{1,2,...,2n\} = \dfrac{ (2n)!}{ (n!)^2} = \dfrac{2n(2n-1)...(n+1)}{n(n-1)...1} > 2^n > 2^{n-1}$

 

Đi thi tiếc thế không làm hoàn chỉnh được bài này, viết được có tới đoạn $P(x)=...$ thì lại lan man đi đâu =))) Không biết có được điểm không nhỉ? 




#674386 CMR: $\sqrt{a + b} + \sqrt{b + c} + \...

Đã gửi bởi Kamii0909 on 15-03-2017 - 22:44 trong Bất đẳng thức và cực trị

Cho a, b, c là các số dương thay đổi và a + b + c = 4. CMR : $\sqrt{a + b} + \sqrt{b + c} + \sqrt{c + a} > 4$

*P/s: Xin lỗi các bạn, mình đã sửa đề :)

KMTTQ, $a \geq b \geq c$

Đpcm $$\Leftrightarrow \sum \sqrt{a+b} \geq 2 \sqrt{a+b+c}$$

$$\Leftrightarrow \sqrt{b+c} \geq \frac{c}{\sqrt{a+b+c}+\sqrt{a+b}} +\frac{b}{\sqrt{a+b+c}+\sqrt{a+c}}$$

Có $$\sqrt{a+b+c}+\sqrt{a+b} \geq \sqrt{a+b+c}+\sqrt{a+c} \geq \sqrt{b+c}$$

Vậy $$R.H.S \leq \frac{b+c}{\sqrt{b+c}}=L.H.S$$

Ta có điều phải chứng minh.

Spoiler




#667825 Chứng minh A,F,I thẳng hàng

Đã gửi bởi Kamii0909 on 09-01-2017 - 22:40 trong Hình học

Nó quanh quanh cấu hình bài hình của USAMO 2008



#658208 $Cho x+y+z\geq 12.Tìm min : P=\frac{x}{\sq...

Đã gửi bởi Kamii0909 on 17-10-2016 - 19:59 trong Bất đẳng thức và cực trị

Bạn viết sigma mình không hiểu gì cả. MÌnh chưa học đến đó

Bạn lên Google tìm đại đi  :wacko:  :wacko: Đằng nào sau này cũng phải biết  :closedeyes: Với cả dùng sigma cho gọn chứ LATEX dài mỏi tay lắm  :D  :D




#658048 $Cho x+y+z\geq 12.Tìm min : P=\frac{x}{\sq...

Đã gửi bởi Kamii0909 on 16-10-2016 - 12:01 trong Bất đẳng thức và cực trị

Bổ đề $3\sum a^{2}b\leq \left ( \sum a \right )\left ( \sum a^{2} \right )$

P=$\sum \frac{x}{\sqrt{y}}= \sum \frac{x^{2}}{x\sqrt{y}}\geq \frac{\left ( \sum x \right )^{2}}{\sum x\sqrt{y}}$

Theo bổ đề ta có $3\sum x\sqrt{y}\leq \left (\sum x \right )\left ( \sum \sqrt{x} \right )$

Dự đoán min=6

Ta phải chứng minh

$\left (\sum x \right )^{2}\geq \left 2( \sum x \right )\left ( \sum \sqrt{x} \right )\Leftrightarrow \sum x\geq 2\sum \sqrt{x}$

Mà theo bđt Cauchy-Schwarz $\sum \sqrt{x}\leq \sqrt{3\sum x}$ nên ta phải chứng minh

$\sqrt{\sum x}\geq 2\sqrt{3}\Leftrightarrow \sum x\geq 12$

Bđt cuối luôn đúng nên ta có min=6 khi x=y=z=4 




#662264 Cho các số thực $a,b,c$ thỏa mãn $a^{2}+b^{2...

Đã gửi bởi Kamii0909 on 17-11-2016 - 22:10 trong Bất đẳng thức và cực trị

v~ cả nhái :D , trích (Poland 1991) đã được giải ở đây bn: http://diendantoanho...e-6#entry660132

Cách giải của bạn hanguyen445 trong phần a không âm chính là cách giải của bài thi trên. 

Bài này khá nổi tiếng, có xuất hiện trong TLCT 10 mà bạn.




#673315 $\sum \frac{a(b+c)}{b^2+bc+c^2}$

Đã gửi bởi Kamii0909 on 03-03-2017 - 01:52 trong Bất đẳng thức - Cực trị

1/CMR:

    a,b,c>0 

          $\sum \frac{a(b+c)}{b^2+bc+c^2}$$\geq 2$

Theo AM-GM $$4(a^2+ab+b^2)(ab+bc+ca) \leq (a+b)^2(a+b+c)^2$$
Bất đẳng thức cần chứng minh đưa về $$\frac{a}{b+c} +\frac{b}{a+c} +\frac{c}{a+b} \geq \frac{(a+b+c)^2}{2(ab+bc+ca)}$$

Hiển nhiên theo C-S. 

Spoiler




#662090 Cho các số thực $a,b,c$ thỏa mãn $a^{2}+b^{2...

Đã gửi bởi Kamii0909 on 15-11-2016 - 22:35 trong Bất đẳng thức và cực trị

Bài này có lẽ "nhái" theo 1 bài thi Châu Âu nhưng bất thành. Thành ra là 1 bài rất khó. Mình nghĩ nó là bài sau:
Cho $x,y,z$ thực thỏa mãn $x^2+y^2+z^2=2$
Chứng minh rằng $x+y+z-xyz \leq 2$.



#668675 Chứng minh di chuyển trên 1 đườnng cố định

Đã gửi bởi Kamii0909 on 17-01-2017 - 17:10 trong Hình học

Cách của mình. Ta sẽ nhắc lại không chứng minh một số kết quả cơ bản sau.
1. Cho tam giác $ABC$ nội tiếp $(O)$. Tiếp tuyến tại $B,C$ của $(O)$ cắt nhau tại $P$. Khi đó $AP$ là đường đối trung đỉnh $A$ của tam giác $ABC$.
2. Trong tam giác $ABC$, đường đối trung đỉnh $A$ cắt $BC$ tại $D$. Khi đó $\frac{DB}{DC}= \frac{AB^2}{AC^2}$
Để xử lí cho gọn, đẹp trước hết ta đi chứng minh bổ đề sau
$\textbf{Bổ đề}$ Cho tam giác $ABC$. Đường đối trung đỉnh $A$ cắt $BC$ tại $Q$. $O,I,J$ lần lượt tâm đường tròn ngoại tiếp tam giác $ABC,AQB,AQC$. Khi đó $OA$ chia đôi $IJ$.
$\textbf{Chứng minh}$
Kẻ $AH,AM$ là đường cao và là trung tuyến của tam giác $ABC$.
Dễ thấy phép vị tự quay tâm $A$ biến $\Delta AIJ \rightarrow \Delta ABC$ biến $H \rightarrow Q$. Mặt khác cũng có $\angle HAM= \angle QAO$ nên biến $ AM \rightarrow AO$. Lại có $AM$ là trung tuyến trong tam giác $ABC$ nên $AO$ là trung tuyến trong tam giác $AIJ$.

$\textbf{Quay lại bài toán}$
Qua $Q$ kẻ đường thẳng $\parallel BC$ cắt $AB,AC$ tại $E,F$. Gọi $I,J$ là tâm đường tròn ngoại tiếp các tam giác $AQE,AQF$.
Theo bổ đề $OA$ chia đôi $IJ$. Ta sẽ chứng minh rằng $O_{1}I=O_{2}J$.
Thật vậy, có $\Delta AIO_{1} \sim \Delta QEB, \Delta AJO_{2} \sim \Delta QFC$ nên ta thu được biến đổi sau.
$$\dfrac{IO_{1}}{JO_{2}}= \frac{IO_{1}}{IA}\cdot \frac{IA}{JA} \cdot \frac{JA}{JO_{2}} = \frac{EB}{QE}\cdot \frac{AB}{AC}\cdot \frac{FQ}{FC}= \frac{AE^2}{AF^2} \cdot \frac{QF}{QE}=1$$
Vậy ta có điều phải chứng minh.



#658168 Đề chọn đội tuyển học sinh giỏi quốc gia Lào Cai 2016-2017

Đã gửi bởi Kamii0909 on 16-10-2016 - 23:58 trong Thi HSG cấp Tỉnh, Thành phố. Olympic 30-4. Đề thi và kiểm tra đội tuyển các cấp.

Chỗ này thiếu r bạn. $gcd\left (3y,y+1 \right)=1$ hoặc $gcd\left (3y,y+1 \right)=3 $ chứ

Nếu $gcd\left (3y,y+1 \right)=3$ thì $(3x+4)^2 \not\vdots 3$, còn $(y+1)(y^2-y+1) \vdots 3$ nên vô lý

Mình có ghi ở trên là $3|y$ rồi nên hiển nhiên là $gcd(y+1,3)=1$ do $y+1$ chia 3 dư 1 




#662754 Cho tam giác ABC với I là tâm đường tròn nội tiếp

Đã gửi bởi Kamii0909 on 22-11-2016 - 21:16 trong Hình học

Nguyên gốc như sau.
$\frac{AI}{AD}=\frac{IH}{ID}$